PT71.S1.Q15 - vitamins a and d

LSATcantwinLSATcantwin Alum Member Sage
edited August 2017 in Logical Reasoning 13286 karma

This is a MMS question that just doesn't make sense to me.
https://7sage.com/lsat_explanations/lsat-71-section-1-question-15/

In the stimulus you are presented with this;

Vitamins A and D can be toxic if you exceed the daily recommended intake.
Some foods, as stated by the manufacturer, have 100% daily recommend value per serving.
Many people over estimate what counts as a serving, sometimes eating 2-3 times more.

I follow it so far.

Here is the issue;

Correct Answer says;

B.) Some people who consume vitamin-fortified foods, exceed the daily intake of Vitamins A and D.

What? How is this supported? Vitamin A and D were never said to be in those foods. It could be Vitamin B and E that are in the foods that people over eat. How do I know some of those foods contain vitamin A and D. It never says anything about that.

The answer choice I picked;

D.) Most people who eat vitamin fortified foods should not take any vitamin supplements.

Isn't this more supported. If many people eat more than they should of vitamin fortified foods, then they need to avoid even more of those vitamins because it's unhealthy. This could be ANY vitamin, A/B/D/E. This seems logically way more supported than AC B. Even with J.Y.'s explanation I still don't understand how B is more supported than D.

Comments

  • AlexAlex Alum Member
    edited August 2017 23929 karma

    @LSATcantwin

    I just looked at this question and I think I understand the difficulty you're running into.

    For answer choice (D) the "most is what makes this AC too strong. We can't infer that most people who eat vitamin fortified foods shouldn't take vitamins. We only can infer from the stim that "many" people overestimate serving size of these vitamin fortified foods. It could theoretically be that most people who eat these foods don't overestimate their serving size.
    I've actually run into this problem with many a bunch of times before I realized that it doesn't actually mean anything as far as the LSAT is concerned. Many people could mean 5 out of 10 or 5 out of 1000.

    Answer choice (B) is far from perfect as we'd expect on a MBT, but it is a MSS so it doesn't have to be air tight. The main reason it is right is because we can infer it with what we're given in the stimulus without having to assume/infer anything further.

    Does that help at all?

  • nathanieljschwartznathanieljschwartz Alum Member
    1723 karma

    I ruled out D for the same reason as @"Alex Divine" mentioned above, but i def see the point @LSATcantwin is pointing out, i also suspect this is a ploy to trip one up. I find that LSAC loves doing this: making the correct AC complete crap but fulfilling the very minimum.
    But the sensitivity to few, many, some, most def points to (B) being the only AC left

  • LSATcantwinLSATcantwin Alum Member Sage
    13286 karma

    @"Alex Divine" said:
    @LSATcantwin

    I just looked at this question and I think I understand the difficulty you're running into.

    For answer choice (D) the "most is what makes this AC too strong. We can't infer that most people who eat vitamin fortified foods shouldn't take vitamins. We only can infer from the stim that "many" people overestimate serving size of these vitamin fortified foods. It could theoretically be that most people who eat these foods don't overestimate their serving size.
    I've actually run into this problem with many a bunch of times before I realized that it doesn't actually mean anything as far as the LSAT is concerned. Many people could mean 5 out of 10 or 5 out of 1000.

    Answer choice (B) is far from perfect as we'd expect on a MBT, but it is a MSS so it doesn't have to be air tight. The main reason it is right is because we can infer it with what we're given in the stimulus without having to assume/infer anything further.

    Does that help at all?

    Yeah, I think "many" is why I picked it. I still don't see how AC B is stronger though.

    B.) Some people who consume vitamin-fortified foods, exceed the daily intake of Vitamins A and D.

    How do we know this? I don't really see how it's supported even a little bit.

    It's saying -

    Some people eat vitamin fortified foods. Of those people, some of them eat more than they should. We don't know what vitamins those foods actually contain, but we will assume some of those foods contain 100% of Vitamin A and D. Then we will assume that the people who eat more than they should, eat the foods that contain Vitamin A and D.

    So we can say that it is supported. Seems like a huge stretch, more than is typical of the LSAT, to me.

  • AlexAlex Alum Member
    23929 karma

    @nathanieljschwartz said:
    I ruled out D for the same reason as @"Alex Divine" mentioned above, but i def see the point @LSATcantwin is pointing out, i also suspect this is a ploy to trip one up. I find that LSAC loves doing this: making the correct AC complete crap but fulfilling the very minimum.
    But the sensitivity to few, many, some, most def points to (B) being the only AC left

    Exactly a ploy to trip us up. When I get down to two answer choices sometimes you just have to find a concrete reason to eliminate and then go with what you have left. That's basically what I did with this question. it was only after I thought about it afterwards that I could articulate a reason why D is wrong and B is right.

  • OlamHafuchOlamHafuch Alum Member
    2326 karma

    Actually, I think that the phrase "these vitamins" has to refer to Vitamins A and D. There is no other way to explain the word "these." It's a very tricky way to use referential phrasing, because it is not a clear way of writing, but it's gotta mean that.

  • AlexAlex Alum Member
    23929 karma

    @LSATcantwin said:

    @"Alex Divine" said:
    @LSATcantwin

    I just looked at this question and I think I understand the difficulty you're running into.

    For answer choice (D) the "most is what makes this AC too strong. We can't infer that most people who eat vitamin fortified foods shouldn't take vitamins. We only can infer from the stim that "many" people overestimate serving size of these vitamin fortified foods. It could theoretically be that most people who eat these foods don't overestimate their serving size.
    I've actually run into this problem with many a bunch of times before I realized that it doesn't actually mean anything as far as the LSAT is concerned. Many people could mean 5 out of 10 or 5 out of 1000.

    Answer choice (B) is far from perfect as we'd expect on a MBT, but it is a MSS so it doesn't have to be air tight. The main reason it is right is because we can infer it with what we're given in the stimulus without having to assume/infer anything further.

    Does that help at all?

    Yeah, I think "many" is why I picked it. I still don't see how AC B is stronger though.

    B.) Some people who consume vitamin-fortified foods, exceed the daily intake of Vitamins A and D.

    How do we know this? I don't really see how it's supported even a little bit.

    It's saying -

    Some people eat vitamin fortified foods. Of those people, some of them eat more than they should. We don't know what vitamins those foods actually contain, but we will assume some of those foods contain 100% of Vitamin A and D. Then we will assume that the people who eat more than they should, eat the foods that contain Vitamin A and D.

    So we can say that it is supported. Seems like a huge stretch, more than is typical of the LSAT, to me.

    AC B is stronger simply because we don't have to assume AS MUCH as we would have to in order to accept D as correct. With D we have to assume most people and for B we have "some" which is ok to infer since we are told "many" in the stimulus... As far as whether or not the foods contain specifically Vitamin A and D. Yeah that part kind of threw me too. I was only confident after I could definitely find reasons to eliminate the other 4. These newer tests sometimes require making inferences I'm not comfortable with...

  • LSATcantwinLSATcantwin Alum Member Sage
    13286 karma

    @uhinberg said:
    Actually, I think that the phrase "these vitamins" has to refer to Vitamins A and D. There is no other way to explain the word "these." It's a very tricky way to use referential phrasing, because it is not a clear way of writing, but it's gotta mean that.

    Mother.....that's what I missed. That changes everything. This test makes me wanna binge eat unhealthy food....lol

  • AlexAlex Alum Member
    23929 karma

    @LSATcantwin said:

    @uhinberg said:
    Actually, I think that the phrase "these vitamins" has to refer to Vitamins A and D. There is no other way to explain the word "these." It's a very tricky way to use referential phrasing, because it is not a clear way of writing, but it's gotta mean that.

    Mother.....that's what I missed. That changes everything. This test makes me wanna binge eat unhealthy food....lol

    Yeah, good catch @uhinberg :)

    Haha I'm going to eat 300% of my daily vitamin intake of A & D!

Sign In or Register to comment.